LSAT and Law School Admissions Forum

Get expert LSAT preparation and law school admissions advice from PowerScore Test Preparation.

 Administrator
PowerScore Staff
  • PowerScore Staff
  • Posts: 8916
  • Joined: Feb 02, 2011
|
#73598
Complete Question Explanation

Weaken. The correct answer choice is (D).

The author seeks to prove that Activate must be effective at promoting energy and mental alertness, based on the premises that they are giving away a month's supply for free and that doing so would not be beneficial if the product didn't work. To weaken the argument, we need and answer that raises at least some doubts about that conclusion. Perhaps the product is not effective, despite those premises? Perhaps the author is wrong about the giveaway being against the company's interests if the product isn't effective, and they do actually stand to gain from the giveaway somehow?

Answer choice (A): This answer has no impact on the argument. It doesn't matter if customers could get all the nutrients in Activate from their food, because the product could still be effective.

Answer choice (B): Another irrelevant answer, as the stimulus had nothing to do with the competition. The issue is only whether the company could somehow benefit from giving away an ineffective product.

Answer choice (C): An attractive answer, perhaps, but only if we assume more information to help it. Answer C would only hurt the argument if customers know that it would take a while to become effective and that they would purchase more of the product in the hope that the effect would eventually appear. As this answer doesn't provide all that extra info, we should not assume it in order to help the answer be better.

Answer choice (D): This is the correct answer choice. If this answer is true (and we are supposed to treat ALL the answers as if they are true), then it would be possible for the makers of the product to benefit even if the product doesn't work and none of those customers buy any more of it. Perhaps the profit they are making on the handling charge is sufficient to cover the cost of the product? While it might not be enough, this answer should at least raise doubts about the conclusion because it shows at least some benefit to the company for shipping the free product.

Answer choice (E): Another irrelevant answer, this one raises a new issue of side effects and does nothing to address the possible benefit to the company in sending out free products that don't work.
 srcline@noctrl.edu
  • Posts: 243
  • Joined: Oct 16, 2015
|
#35313
Hello,

So I was in b/w two a.c. on this B and D and ended up choosing B. So this asks us to weaken the conclusion that:

"clearly, Acitivate must be effective since otherwise it would not be in the company's interest to make such an offer.

So we're trying to weaken its effectiveness as well as the financial aspect right? Too me B weakens the argument b.c if there are dairy supplements that are equal in effectiveness as well as cost less Activite is not justified in making the offer. I know that B doesn't weaken the point about effectiveness but neither does D. If you charge a handling fee ...then the offer isnt really free cause they're still making money off of it but how does this relate to the matter about effectiveness.

Is D better because its more specific?

Thankyou
Sarah
 Emily Haney-Caron
PowerScore Staff
  • PowerScore Staff
  • Posts: 577
  • Joined: Jan 12, 2012
|
#35385
Hi Sarah,

I think it might help for you to go back to this one and read it over again a few times. We're trying to weaken the conclusion that the company would not benefit from giving a free trial on a product that doesn't work, and so the product must work. My prephrase here is that I want an answer that explains why the company benefits from giving a free trial of a bad product. We find that in D: the company benefits from giving the free trial even if no one ever buys the product, because they make money from the "free trial." Therefore, even if the product is terrible, if they can get lots of people to pay for a "free" trial, they'll still make money.

B doesn't actually challenge the conclusion at all. Remember, we're not trying to show the product isn't effective (and, anyway, B doesn't do that; it doesn't say Activite doesn't work, just that cheaper things work just as well). Instead, we're trying to challenge the assumption that giving a free trial means it has to be effective. That's a small distinction, but an important one here! B doesn't speak to that point in any way, and is therefore irrelevant.
 dbpk
  • Posts: 16
  • Joined: May 07, 2017
|
#39040
hello!

For answer choice C, would you say the use of the phrase "most dietary supplements" makes this a poor strengthen answer because it creates ambiguity as to whether it is relevant to Activite at all? Should we avoid answers with "most x" for strengthen questions?

Additionally, I was wary of answer choice D because it only mentions that handling fees are higher than the costs of packing and shipping but this excludes so many other potentially larger fees like the cost to manufacture the actual product, labor, etc. I suppose it is possible that the handling fee is so high that it covers all of these costs so the makers of Activite make a profit but one would definitely not be able to infer with certainty that this is the case. Is it enough that an answer choice COULD strengthen the argument for it to be correct?

Thank you very much!
 Adam Tyson
PowerScore Staff
  • PowerScore Staff
  • Posts: 5153
  • Joined: Apr 14, 2011
|
#39383
First off, dbpk, this is not a Strengthen question but a Weaken question. Your analysis seems like you understood that, so I am assuming that was just a typo.

While you are correct that answer D doesn't PROVE that the product is not effective, because there may be many other costs associated with producing the product and thus still no benefit to giving it away, the goal of a weaken question isn't to PROVE that the conclusion is wrong, but merely to introduce some element of doubt. If D is true, then MAYBE they are making money on the product, gaining a benefit even without the product being effective? That's all we need to weaken the conclusion here.

Answer C, about most dietary supplements, tells us nothing about THIS dietary supplement. Maybe it's in the majority, maybe it's in the minority. This gives us no reason to doubt the effectiveness of the product. Maybe it will work within the first month, or maybe people will need to buy another month's worth in order to get the cumulative effect. We want to weaken the claim that the product must be effective, and this just does nothing for us. It's not about how long it takes to become effective, but about whether it EVER is effective. The argument says it must be, and we want to suggest that it might not be. Suggesting that they can profit despite it being ineffective is what gives us that moment of doubt.

I hope that clears things up some for you!
 chzhou
  • Posts: 2
  • Joined: Aug 31, 2018
|
#57036
Um.... why can't we assume that when they say "free" that they mean "free?" this means even if there were handling costs, they weren't imposed on those getting the "free" monthly supply. how are we to assume from D that just because Activite charges handling fees that they did in this instance? Heck, they probably charge for the product, too, but they didn't in this "free" monthly trial. I would buy Answer D if it said that they charged a handling fee for shipping the free stuff, but they don't specify that, and I don't want to go to LSAT prison for making an illicit assumption.
 Rachael Wilkenfeld
PowerScore Staff
  • PowerScore Staff
  • Posts: 1358
  • Joined: Dec 15, 2011
|
#57352
Hi Chzhou,

This requires looking very closely at what was offered by the company. They don't charge for the first month's "supply." That's different than saying that the entire order is free. It just means the supply, or physical substance ordered, doesn't cost money. There could still be shipping costs, or as we see in answer choice (D), handling costs, that raise the overall cost of the order.

Hope that helps!
Rachael
 kenlars5
  • Posts: 21
  • Joined: Oct 27, 2020
|
#82374
Hi there,

I struggled with this problem for a while until I finally think I got a grasp on it but I just wanted to talk about my thinking and see if the way I approached is correct & the best way!

So for this one I took the argument to be causal:

it’s effective —> that’s why they sent out 1 month free supply

So to weaken I was looking for an answer that would offer an alternative explanation/reason for why they would send out 1 month supply even if it wasn’t effective... at first I chose C because my thinking was well if a month is not a sufficient length of time for most dietary supplements to be fully effective than they must have sent it for another reason - looking back now I realize choosing such an answer would require you to assume some things.. 1. That Activite knows that most dietary supplements aren’t fully effective in one month (if they didn’t maybe they sent it because they thought it would prove to be effective in that time). 2. That Activite is a product that is included in the “most” that the answer talks about (just because most aren’t fully effective doesn’t mean Activite isn’t one that IS). And even if both of those are true, it still doesn’t give us an alternative reason that would make us doubt that being effective is the reason they send one month free.. so it really has no effective on the argument without assuming some things.

However D, gives a alternative reason/interest for why Activite would send 1 month free to new customers. It’s not that it’s effective, it’s because they they still make money from the handling, as handling costs are even more than what it costs them to pack and ship their product. If so that’s a different interest for why they do it.

I’d appreciate any insight if this was a good approach or what I could’ve thought/done to make this question easier. Looking back I honestly think I just failed to recognize what D was saying as I took the stimulus to say the first month was was completely free for the customers, which yes the first month of products was free but doesn’t mean you don’t have to pay handling costs which is separate from the products costs. Besides this misinterpretation I really don’t think this question should’ve been that difficult. I look forward to hearing back,
Thanks! :)
 Robert Carroll
PowerScore Staff
  • PowerScore Staff
  • Posts: 1787
  • Joined: Dec 06, 2013
|
#82548
ken,

I think your reasoning is pretty good. This is a tough question, need the end of the section, and the "handling fee" issue in answer choice (D) is probably something most people are not going to think about until it comes up, so their prephrases won't match it. Realizing that all the other answers don't work, and that answer choice (D) is a way to weaken the argument (whether you've thought about it before seeing it or not) is, in my opinion, a perfectly fine way to get this one. It's a bit time consuming, but, again, it's a late, tough question. I'm comfortable taking extra time on harder questions, as long as I can save time on earlier ones.

I have a couple problems with answer choice (C) that I didn't see addressed in your post:

1. Activite doesn't have to demonstrate efficacy in the first month for people to want to buy it after the free month passes. I'll use an example from my own life - I take fish oil pills every day. Some people much smarter than I tell me fish oil is good for health. I couldn't honestly tell you that I notice a difference after taking a month of them, but, when I run out of this current batch, you know what I'm going to do? Go on Amazon and buy some more. Similarly, even if Activite doesn't do anything after a month, people aren't necessarily perceiving its inefficacy. So this doesn't give us a particularly good reason to think people will discontinue it after a month,.

2. The phrase "fully effective" in the answer gives me serious pause. If Activite isn't fully effective after a month, it could still be effective enough for people to want to get more of it. It's unclear how effective it would have to be for people to drop it.

I think the issues you noted are good as well. And after analyzing answer choice (C), I want to circle back to what we're doing - trying to show that Activite might not be effective just because of what the company is doing to sell it. If that sales strategy can be cost-effective even without the efficacy of the product, then the argument is weakened. Answer choice (D) is showing how they could make money in a way completely divorced from the efficacy of the product.

Robert Carroll
 kenlars5
  • Posts: 21
  • Joined: Oct 27, 2020
|
#82811
Ahh, thank you Robert! That helped a lot!

Get the most out of your LSAT Prep Plus subscription.

Analyze and track your performance with our Testing and Analytics Package.